LSAT and Law School Admissions Forum

Get expert LSAT preparation and law school admissions advice from PowerScore Test Preparation.

User avatar
 Dave Killoran
PowerScore Staff
  • PowerScore Staff
  • Posts: 5853
  • Joined: Mar 25, 2011
|
#79285
Complete Question Explanation
(The complete setup for this game can be found here: lsat/viewtopic.php?t=17034)

The correct answer choice is (A).

From our analysis of the game, we know that M must connect to either J or N, but not both. The easiest way to force M to connect to N is to eliminate J as a possible connecting chalet. The easiest way to eliminate J from contention is to connect it so another chalet, thus connecting it to two chalets, thereby closing it off to further connections. Answer choice (A), which connects K and J, would maximize the number of connections J has, and would thus force M to connect to N. Answer choice (A) is correct.

Note that, once you determine that closing off J is the correct way to achieve the result desired in the question stem, you should immediately scan the answer choices for J. Only answer choice (A) contains J.
 rameday
  • Posts: 94
  • Joined: May 07, 2014
|
#15762
Hello,

So I am a bit confused as to this question.

So I got the 6 templates. And for this one I get that we need to be looking at the 3 M-N templates. I put answer E because that was common to all three templates. Answer choice A the correct answer was only common to 1 template. What Jon did in the module was that he saw if we could achieve the K-J connection that the answer choice calls for without having that M-N connection at the same time. I get how that got him to the correct answer A and allowed him to eliminate E. However, what I don't get is why it is ok that the k-J connection needs to be in only one of the three templates as opposed to all three simultaneously. I thought it had to be in all three simultaneously.

A
 Jon Denning
PowerScore Staff
  • PowerScore Staff
  • Posts: 904
  • Joined: Apr 11, 2011
|
#15779
Hey A,

Think of what you're trying to accomplish here: We want to be forced to connect M and N. Answer choice A makes that happen by giving J two connections (the max), meaning M cannot be connected to J and must go to N.

Answer choice E, on the other hand, doesn't force M and N together, as M could still connect to J instead. Follow MJNKLO as single connections with E and you're fine, no M-N.

Thanks!

Get the most out of your LSAT Prep Plus subscription.

Analyze and track your performance with our Testing and Analytics Package.